[Math] Is the Fourier Transform in $L^1$

fourier analysisfourier transform

Let $f$ be defined $$f = \begin{cases}
\frac{1}{(t+1)^3}, & t > 0 \\
0, & t \leq 0
\end{cases}$$

Is the Fourier transform of $f$ in $L^1(\mathbb{R})$?

I know that the decay of the Fourier transform can be induced from smoothness of the original function, but I don't see how this is used here, as $f$ is not continuous at $0$.

I think this may have something to do with the inverse Fourier transform: if the Fourier transform is in $L^1$, then it itself has a Fourier transform which has to be continuous everywhere.

Best Answer

We know that $f \in L^1(\mathbb R)$, then $\hat{f}$ is bounded and continuous. If $\hat{f} \in L^1(\mathbb{R})$ then almost everywhere $$f(x)=\dfrac{1}{2\pi}\int_{\mathbb R}e^{i\xi x}\hat{f}(\xi)\ \mathrm{d}\xi$$ This means that $f$ is equal almost everywhere to a continuous function $g$. We find a contradiction around $x=0$ because we know that there exists $\delta > 0$ such that for $x \in [-\delta, \delta], \ g(x) \in [g(0)-0.1, g(0)+0.1]$. Therefore for almost every $x \in [-\delta, \delta], \ f(x) \in [g(0)-0.1, g(0)+0.1]$, which is not the case.

Related Question